Prove there exists a measurable subset $E' subset E$ such that $f$ is almost everywhere of one sign on $E'$.












2












$begingroup$


Let $f in Lleft(mathbb{R}right)$ where $L$ denotes Lebesgue integrable space of functions. Suppose there exists a subset $E subset mathbb{R}$ such that $m(E) > 0$ and $f(x) neq 0$ for all $x in E$. Prove there exists a measurable subset $E' subset E$ such $m(E') > 0$ and $f$ is almost everywhere of one sign on $E'$.





My (weak) attempt:



Let $E_alpha = {E setminus A: m(E) = m(E setminus A)}$.... I don't know. Rewrote this proof like 10 times with no insight. So now I'm just throwing it out there. Thank you.





I think all I need is a hint to start it.










share|cite|improve this question









$endgroup$

















    2












    $begingroup$


    Let $f in Lleft(mathbb{R}right)$ where $L$ denotes Lebesgue integrable space of functions. Suppose there exists a subset $E subset mathbb{R}$ such that $m(E) > 0$ and $f(x) neq 0$ for all $x in E$. Prove there exists a measurable subset $E' subset E$ such $m(E') > 0$ and $f$ is almost everywhere of one sign on $E'$.





    My (weak) attempt:



    Let $E_alpha = {E setminus A: m(E) = m(E setminus A)}$.... I don't know. Rewrote this proof like 10 times with no insight. So now I'm just throwing it out there. Thank you.





    I think all I need is a hint to start it.










    share|cite|improve this question









    $endgroup$















      2












      2








      2





      $begingroup$


      Let $f in Lleft(mathbb{R}right)$ where $L$ denotes Lebesgue integrable space of functions. Suppose there exists a subset $E subset mathbb{R}$ such that $m(E) > 0$ and $f(x) neq 0$ for all $x in E$. Prove there exists a measurable subset $E' subset E$ such $m(E') > 0$ and $f$ is almost everywhere of one sign on $E'$.





      My (weak) attempt:



      Let $E_alpha = {E setminus A: m(E) = m(E setminus A)}$.... I don't know. Rewrote this proof like 10 times with no insight. So now I'm just throwing it out there. Thank you.





      I think all I need is a hint to start it.










      share|cite|improve this question









      $endgroup$




      Let $f in Lleft(mathbb{R}right)$ where $L$ denotes Lebesgue integrable space of functions. Suppose there exists a subset $E subset mathbb{R}$ such that $m(E) > 0$ and $f(x) neq 0$ for all $x in E$. Prove there exists a measurable subset $E' subset E$ such $m(E') > 0$ and $f$ is almost everywhere of one sign on $E'$.





      My (weak) attempt:



      Let $E_alpha = {E setminus A: m(E) = m(E setminus A)}$.... I don't know. Rewrote this proof like 10 times with no insight. So now I'm just throwing it out there. Thank you.





      I think all I need is a hint to start it.







      measure-theory lebesgue-measure






      share|cite|improve this question













      share|cite|improve this question











      share|cite|improve this question




      share|cite|improve this question










      asked Dec 3 '18 at 21:11









      ZduffZduff

      1,663920




      1,663920






















          1 Answer
          1






          active

          oldest

          votes


















          2












          $begingroup$

          Hint: take $P = {f > 0}$ and $N= {f < 0}$. Now,



          $$
          E = (E cap P) sqcup (E cap N).
          $$



          Certainly $f$ maintains its sign in each of these. Can they both be of measure zero?






          share|cite|improve this answer









          $endgroup$













          • $begingroup$
            Ah, thank you. But is it easy to show these two sets are measurable?
            $endgroup$
            – Zduff
            Dec 4 '18 at 2:51






          • 1




            $begingroup$
            Yes, intersection of measurable sets is measurable and $f in L(mathbb{R})$ implies measurability. Hence both level sets are measurable.
            $endgroup$
            – Guido A.
            Dec 4 '18 at 13:08










          • $begingroup$
            So if $f$ is Lebesgue integrable, then its level sets are always measurable? What is that theorem called, or do you possibly have a link?
            $endgroup$
            – Zduff
            Dec 4 '18 at 14:51










          • $begingroup$
            You can find surely find it on Zygmund's Measure and Integral. I can give you a sketch of the proof: in the book it is proven (in the chapter on Fubini's theorem) that if $E subset mathbb{R}^{n_1 + n_2}$ is measurable, then $E^y := {x : (x,y) in E }$ is measurable a.e. $y in mathbb{R}^{n_2}$. Since the integral can be define as the measure of the set under the graphic of $f$, the fact that $f$ is integrable tells us that this set $R(f,E)$ is measurable and so $R(f,E)^y = { f > y}$ is measurable a.e. $y in mathbb{R}$.
            $endgroup$
            – Guido A.
            Dec 4 '18 at 20:09










          • $begingroup$
            (cont.) Since the set where the level sets are not measureable is of measure zero, the set where they are measurable must be dense, and so $f$ is measurable (this last result is also proved in the same book).
            $endgroup$
            – Guido A.
            Dec 4 '18 at 20:10











          Your Answer





          StackExchange.ifUsing("editor", function () {
          return StackExchange.using("mathjaxEditing", function () {
          StackExchange.MarkdownEditor.creationCallbacks.add(function (editor, postfix) {
          StackExchange.mathjaxEditing.prepareWmdForMathJax(editor, postfix, [["$", "$"], ["\\(","\\)"]]);
          });
          });
          }, "mathjax-editing");

          StackExchange.ready(function() {
          var channelOptions = {
          tags: "".split(" "),
          id: "69"
          };
          initTagRenderer("".split(" "), "".split(" "), channelOptions);

          StackExchange.using("externalEditor", function() {
          // Have to fire editor after snippets, if snippets enabled
          if (StackExchange.settings.snippets.snippetsEnabled) {
          StackExchange.using("snippets", function() {
          createEditor();
          });
          }
          else {
          createEditor();
          }
          });

          function createEditor() {
          StackExchange.prepareEditor({
          heartbeatType: 'answer',
          autoActivateHeartbeat: false,
          convertImagesToLinks: true,
          noModals: true,
          showLowRepImageUploadWarning: true,
          reputationToPostImages: 10,
          bindNavPrevention: true,
          postfix: "",
          imageUploader: {
          brandingHtml: "Powered by u003ca class="icon-imgur-white" href="https://imgur.com/"u003eu003c/au003e",
          contentPolicyHtml: "User contributions licensed under u003ca href="https://creativecommons.org/licenses/by-sa/3.0/"u003ecc by-sa 3.0 with attribution requiredu003c/au003e u003ca href="https://stackoverflow.com/legal/content-policy"u003e(content policy)u003c/au003e",
          allowUrls: true
          },
          noCode: true, onDemand: true,
          discardSelector: ".discard-answer"
          ,immediatelyShowMarkdownHelp:true
          });


          }
          });














          draft saved

          draft discarded


















          StackExchange.ready(
          function () {
          StackExchange.openid.initPostLogin('.new-post-login', 'https%3a%2f%2fmath.stackexchange.com%2fquestions%2f3024679%2fprove-there-exists-a-measurable-subset-e-subset-e-such-that-f-is-almost-ev%23new-answer', 'question_page');
          }
          );

          Post as a guest















          Required, but never shown

























          1 Answer
          1






          active

          oldest

          votes








          1 Answer
          1






          active

          oldest

          votes









          active

          oldest

          votes






          active

          oldest

          votes









          2












          $begingroup$

          Hint: take $P = {f > 0}$ and $N= {f < 0}$. Now,



          $$
          E = (E cap P) sqcup (E cap N).
          $$



          Certainly $f$ maintains its sign in each of these. Can they both be of measure zero?






          share|cite|improve this answer









          $endgroup$













          • $begingroup$
            Ah, thank you. But is it easy to show these two sets are measurable?
            $endgroup$
            – Zduff
            Dec 4 '18 at 2:51






          • 1




            $begingroup$
            Yes, intersection of measurable sets is measurable and $f in L(mathbb{R})$ implies measurability. Hence both level sets are measurable.
            $endgroup$
            – Guido A.
            Dec 4 '18 at 13:08










          • $begingroup$
            So if $f$ is Lebesgue integrable, then its level sets are always measurable? What is that theorem called, or do you possibly have a link?
            $endgroup$
            – Zduff
            Dec 4 '18 at 14:51










          • $begingroup$
            You can find surely find it on Zygmund's Measure and Integral. I can give you a sketch of the proof: in the book it is proven (in the chapter on Fubini's theorem) that if $E subset mathbb{R}^{n_1 + n_2}$ is measurable, then $E^y := {x : (x,y) in E }$ is measurable a.e. $y in mathbb{R}^{n_2}$. Since the integral can be define as the measure of the set under the graphic of $f$, the fact that $f$ is integrable tells us that this set $R(f,E)$ is measurable and so $R(f,E)^y = { f > y}$ is measurable a.e. $y in mathbb{R}$.
            $endgroup$
            – Guido A.
            Dec 4 '18 at 20:09










          • $begingroup$
            (cont.) Since the set where the level sets are not measureable is of measure zero, the set where they are measurable must be dense, and so $f$ is measurable (this last result is also proved in the same book).
            $endgroup$
            – Guido A.
            Dec 4 '18 at 20:10
















          2












          $begingroup$

          Hint: take $P = {f > 0}$ and $N= {f < 0}$. Now,



          $$
          E = (E cap P) sqcup (E cap N).
          $$



          Certainly $f$ maintains its sign in each of these. Can they both be of measure zero?






          share|cite|improve this answer









          $endgroup$













          • $begingroup$
            Ah, thank you. But is it easy to show these two sets are measurable?
            $endgroup$
            – Zduff
            Dec 4 '18 at 2:51






          • 1




            $begingroup$
            Yes, intersection of measurable sets is measurable and $f in L(mathbb{R})$ implies measurability. Hence both level sets are measurable.
            $endgroup$
            – Guido A.
            Dec 4 '18 at 13:08










          • $begingroup$
            So if $f$ is Lebesgue integrable, then its level sets are always measurable? What is that theorem called, or do you possibly have a link?
            $endgroup$
            – Zduff
            Dec 4 '18 at 14:51










          • $begingroup$
            You can find surely find it on Zygmund's Measure and Integral. I can give you a sketch of the proof: in the book it is proven (in the chapter on Fubini's theorem) that if $E subset mathbb{R}^{n_1 + n_2}$ is measurable, then $E^y := {x : (x,y) in E }$ is measurable a.e. $y in mathbb{R}^{n_2}$. Since the integral can be define as the measure of the set under the graphic of $f$, the fact that $f$ is integrable tells us that this set $R(f,E)$ is measurable and so $R(f,E)^y = { f > y}$ is measurable a.e. $y in mathbb{R}$.
            $endgroup$
            – Guido A.
            Dec 4 '18 at 20:09










          • $begingroup$
            (cont.) Since the set where the level sets are not measureable is of measure zero, the set where they are measurable must be dense, and so $f$ is measurable (this last result is also proved in the same book).
            $endgroup$
            – Guido A.
            Dec 4 '18 at 20:10














          2












          2








          2





          $begingroup$

          Hint: take $P = {f > 0}$ and $N= {f < 0}$. Now,



          $$
          E = (E cap P) sqcup (E cap N).
          $$



          Certainly $f$ maintains its sign in each of these. Can they both be of measure zero?






          share|cite|improve this answer









          $endgroup$



          Hint: take $P = {f > 0}$ and $N= {f < 0}$. Now,



          $$
          E = (E cap P) sqcup (E cap N).
          $$



          Certainly $f$ maintains its sign in each of these. Can they both be of measure zero?







          share|cite|improve this answer












          share|cite|improve this answer



          share|cite|improve this answer










          answered Dec 3 '18 at 21:25









          Guido A.Guido A.

          7,5261730




          7,5261730












          • $begingroup$
            Ah, thank you. But is it easy to show these two sets are measurable?
            $endgroup$
            – Zduff
            Dec 4 '18 at 2:51






          • 1




            $begingroup$
            Yes, intersection of measurable sets is measurable and $f in L(mathbb{R})$ implies measurability. Hence both level sets are measurable.
            $endgroup$
            – Guido A.
            Dec 4 '18 at 13:08










          • $begingroup$
            So if $f$ is Lebesgue integrable, then its level sets are always measurable? What is that theorem called, or do you possibly have a link?
            $endgroup$
            – Zduff
            Dec 4 '18 at 14:51










          • $begingroup$
            You can find surely find it on Zygmund's Measure and Integral. I can give you a sketch of the proof: in the book it is proven (in the chapter on Fubini's theorem) that if $E subset mathbb{R}^{n_1 + n_2}$ is measurable, then $E^y := {x : (x,y) in E }$ is measurable a.e. $y in mathbb{R}^{n_2}$. Since the integral can be define as the measure of the set under the graphic of $f$, the fact that $f$ is integrable tells us that this set $R(f,E)$ is measurable and so $R(f,E)^y = { f > y}$ is measurable a.e. $y in mathbb{R}$.
            $endgroup$
            – Guido A.
            Dec 4 '18 at 20:09










          • $begingroup$
            (cont.) Since the set where the level sets are not measureable is of measure zero, the set where they are measurable must be dense, and so $f$ is measurable (this last result is also proved in the same book).
            $endgroup$
            – Guido A.
            Dec 4 '18 at 20:10


















          • $begingroup$
            Ah, thank you. But is it easy to show these two sets are measurable?
            $endgroup$
            – Zduff
            Dec 4 '18 at 2:51






          • 1




            $begingroup$
            Yes, intersection of measurable sets is measurable and $f in L(mathbb{R})$ implies measurability. Hence both level sets are measurable.
            $endgroup$
            – Guido A.
            Dec 4 '18 at 13:08










          • $begingroup$
            So if $f$ is Lebesgue integrable, then its level sets are always measurable? What is that theorem called, or do you possibly have a link?
            $endgroup$
            – Zduff
            Dec 4 '18 at 14:51










          • $begingroup$
            You can find surely find it on Zygmund's Measure and Integral. I can give you a sketch of the proof: in the book it is proven (in the chapter on Fubini's theorem) that if $E subset mathbb{R}^{n_1 + n_2}$ is measurable, then $E^y := {x : (x,y) in E }$ is measurable a.e. $y in mathbb{R}^{n_2}$. Since the integral can be define as the measure of the set under the graphic of $f$, the fact that $f$ is integrable tells us that this set $R(f,E)$ is measurable and so $R(f,E)^y = { f > y}$ is measurable a.e. $y in mathbb{R}$.
            $endgroup$
            – Guido A.
            Dec 4 '18 at 20:09










          • $begingroup$
            (cont.) Since the set where the level sets are not measureable is of measure zero, the set where they are measurable must be dense, and so $f$ is measurable (this last result is also proved in the same book).
            $endgroup$
            – Guido A.
            Dec 4 '18 at 20:10
















          $begingroup$
          Ah, thank you. But is it easy to show these two sets are measurable?
          $endgroup$
          – Zduff
          Dec 4 '18 at 2:51




          $begingroup$
          Ah, thank you. But is it easy to show these two sets are measurable?
          $endgroup$
          – Zduff
          Dec 4 '18 at 2:51




          1




          1




          $begingroup$
          Yes, intersection of measurable sets is measurable and $f in L(mathbb{R})$ implies measurability. Hence both level sets are measurable.
          $endgroup$
          – Guido A.
          Dec 4 '18 at 13:08




          $begingroup$
          Yes, intersection of measurable sets is measurable and $f in L(mathbb{R})$ implies measurability. Hence both level sets are measurable.
          $endgroup$
          – Guido A.
          Dec 4 '18 at 13:08












          $begingroup$
          So if $f$ is Lebesgue integrable, then its level sets are always measurable? What is that theorem called, or do you possibly have a link?
          $endgroup$
          – Zduff
          Dec 4 '18 at 14:51




          $begingroup$
          So if $f$ is Lebesgue integrable, then its level sets are always measurable? What is that theorem called, or do you possibly have a link?
          $endgroup$
          – Zduff
          Dec 4 '18 at 14:51












          $begingroup$
          You can find surely find it on Zygmund's Measure and Integral. I can give you a sketch of the proof: in the book it is proven (in the chapter on Fubini's theorem) that if $E subset mathbb{R}^{n_1 + n_2}$ is measurable, then $E^y := {x : (x,y) in E }$ is measurable a.e. $y in mathbb{R}^{n_2}$. Since the integral can be define as the measure of the set under the graphic of $f$, the fact that $f$ is integrable tells us that this set $R(f,E)$ is measurable and so $R(f,E)^y = { f > y}$ is measurable a.e. $y in mathbb{R}$.
          $endgroup$
          – Guido A.
          Dec 4 '18 at 20:09




          $begingroup$
          You can find surely find it on Zygmund's Measure and Integral. I can give you a sketch of the proof: in the book it is proven (in the chapter on Fubini's theorem) that if $E subset mathbb{R}^{n_1 + n_2}$ is measurable, then $E^y := {x : (x,y) in E }$ is measurable a.e. $y in mathbb{R}^{n_2}$. Since the integral can be define as the measure of the set under the graphic of $f$, the fact that $f$ is integrable tells us that this set $R(f,E)$ is measurable and so $R(f,E)^y = { f > y}$ is measurable a.e. $y in mathbb{R}$.
          $endgroup$
          – Guido A.
          Dec 4 '18 at 20:09












          $begingroup$
          (cont.) Since the set where the level sets are not measureable is of measure zero, the set where they are measurable must be dense, and so $f$ is measurable (this last result is also proved in the same book).
          $endgroup$
          – Guido A.
          Dec 4 '18 at 20:10




          $begingroup$
          (cont.) Since the set where the level sets are not measureable is of measure zero, the set where they are measurable must be dense, and so $f$ is measurable (this last result is also proved in the same book).
          $endgroup$
          – Guido A.
          Dec 4 '18 at 20:10


















          draft saved

          draft discarded




















































          Thanks for contributing an answer to Mathematics Stack Exchange!


          • Please be sure to answer the question. Provide details and share your research!

          But avoid



          • Asking for help, clarification, or responding to other answers.

          • Making statements based on opinion; back them up with references or personal experience.


          Use MathJax to format equations. MathJax reference.


          To learn more, see our tips on writing great answers.




          draft saved


          draft discarded














          StackExchange.ready(
          function () {
          StackExchange.openid.initPostLogin('.new-post-login', 'https%3a%2f%2fmath.stackexchange.com%2fquestions%2f3024679%2fprove-there-exists-a-measurable-subset-e-subset-e-such-that-f-is-almost-ev%23new-answer', 'question_page');
          }
          );

          Post as a guest















          Required, but never shown





















































          Required, but never shown














          Required, but never shown












          Required, but never shown







          Required, but never shown

































          Required, but never shown














          Required, but never shown












          Required, but never shown







          Required, but never shown







          Popular posts from this blog

          Biblatex bibliography style without URLs when DOI exists (in Overleaf with Zotero bibliography)

          ComboBox Display Member on multiple fields

          Is it possible to collect Nectar points via Trainline?